Vous êtes sur la page 1sur 7

Exercices de Mathématiques

Partie entière, Borne sup et inf


Énoncés

Énoncés des exercices

Exercice 1 [ Indication ] [ Correction ]


Résoudre E(2x − 1) = E(x − 4) dans IR.

Exercice 2 [ Indication ] [ Correction ]


Etudier lim [un ] si lim un = `.
∞ ∞

Exercice 3 [ Indication ] [ Correction ]


1
Calculer lim ([x] + [2x] + · · · + [nx]).
n→∞ n2

Exercice 4 [ Indication ] [ Correction ]


Soient x un réel, et n un entier naturel non nul.
On note t 7→ [t] l’opération “partie entière”.    
x+k [x] + k
1. Montrer que pour tout entier relatif k, = .
n n
n−1  
X x+k
2. Montrer que = [x].
k=0
n

Exercice 5 [ Indication ] [ Correction ]


Montrer que tout rationnel r de [0, 1[ s’écrit d’une manière unique :
a2 a3 an
r= + + ··· + + ···
2! 3! n!
les entiers ai (nuls à partir d’un certain rang) vérifiant 0 ≤ ai < i.
5
Mettre sous cette forme le rationnel .
7

Exercice 6 [ Indication ] [ Correction ]


n o
Calculer λ = inf sup |x2 + tx| .
t∈IR x∈[0,1]

Page 1 Jean-Michel Ferrard www.klubprepa.net EduKlub


c S.A.
Tous droits de l’auteur des œuvres réservés. Sauf autorisation, la reproduction ainsi que toute utilisation des œuvres autre que la consultation
individuelle et privée sont interdites.
Exercices de Mathématiques
Partie entière, Borne sup et inf
Indications, résultats

Indications ou résultats
Indication pour l’exercice 1 [ Retour à l’énoncé ]
Noter k = [x], et discuter suivant le placement de x par rapport à k + 12 .
L’ensemble des solutions est l’intervalle [− 27 , − 52 [.

Indication pour l’exercice 2 [ Retour à l’énoncé ]


– Si ` = ±∞, alors lim [un ] = ` :
n→+∞

– Si ` n’est pas entier, la suite ([un ]) est stationnaire donc convergente en [`].
– Si ` est un entier, alors plusieurs cas sont possibles. . .,

Indication pour l’exercice 3 [ Retour à l’énoncé ]


x
Utiliser kx ≤ [kx] < kx + 1. On trouve lim un (x) = .
n→+∞ 2

Indication pour l’exercice 4 [ Retour à l’énoncé ]


 
x+k
1. Si q = , qn − k ≤ x < (q + 1)n − k ⇒ qn − k ≤ [x] < (q + 1)n − k ⇒ . . .
n
2. On suppose d’abord que x ∈ Z. Soit x = qn + r la division de x par n.
Pour k ∈ {0, . . . , n − 1}, discuter suivant la position de r + k par rapport à n.
Si x est un réel quelconque, utiliser la question (1).

Indication pour l’exercice 5 [ Retour à l’énoncé ]


n−1
P ak P ak
– On commence par l’unicité des (an ). On note : sn = et rn = .
k=2 k! k≥n+1 k!
Prouver an = [n!(r − sn )] = [n!r] − n!sn .
– Réciproquement, on a 0 ≤ an ≤ n − 1 (récurrence). Soit N ∈ IN tel que N !r soit entier.
Prouver que pour tout n ≥ N , on a an+1 = 0, donc sn+1 = r.
5 5 1 1 1 4 2
– La décomposition de r = est : = + + + + .
7 7 2! 3! 4! 6! 7!

Indication pour l’exercice 6 [ Retour à l’énoncé ]


– Traiter le cas t ≥ 0.
– Si t < 0, étudier x 7→ |x(x + t)|.
Il apparaı̂t alors les cas t ≤ −2, −2 ≤ t ≤ −1, −1 ≤ t ≤ 0.
– La borne inférieure λ est obtenue en supposant −1 ≤ t ≤ 0.
√ √
Plus précisément, elle est obtenue quand t = 2 − 2 2 et elle vaut 3 − 2 2.

Page 2 Jean-Michel Ferrard www.klubprepa.net EduKlub


c S.A.
Tous droits de l’auteur des œuvres réservés. Sauf autorisation, la reproduction ainsi que toute utilisation des œuvres autre que la consultation
individuelle et privée sont interdites.
Exercices de Mathématiques
Partie entière, Borne sup et inf
Corrigés

Corrigés des exercices

Corrigé de l’exercice 1 [ Retour à l’énoncé ]


Posons k = [x]. On a k ≤ x < k + 1.
1
– Si k ≤ x < k + 2
alors 2k ≤ 2x < 2k + 1 et [2x] = 2k.
L’équation équivaut alors à 2k − 1 = k − 4, c’est-à-dire k = −3.
On obtient ainsi les solutions x ∈ [−3, − 25 [.
1
– Si k + 2
≤ x < k + 1 alors 2k + 1 ≤ 2x < 2k + 2 et [2x] = 2k + 1.
L’équation équivaut alors à 2k = k − 4, c’est-à-dire k = −4.
On obtient ainsi les solutions x ∈ [− 27 , −3[.
– Finalement, l’ensemble des solutions est l’intervalle [− 72 , − 52 [.

Corrigé de l’exercice 2 [ Retour à l’énoncé ]

– Si ` = ±∞, alors lim [un ] = ` : C’est une conséquence de un − 1 < [un ] ≤ un .


n→+∞

Dans la suite, on suppose que ` est un réel.


– Supposons que ` ne soit pas entier. Notons k = [`]. On a k < ` < k + 1.
Soit ε > 0 tel que k < ` − ε < ` + ε < k + 1.
Il existe un entier n0 tel que n ≥ n0 ⇒ ` − ε ≤ un ≤ ` + ε.
Pour tout n ≥ n0 , on a donc : [un ] = k = [`].
Ainsi la suite ([un ]) est stationnaire en [`] donc convergente en ce point.
– Si ` est un entier, alors plusieurs cas sont possibles :
 Si ∃n0 tel que n ≥ n0 ⇒ un ≥ `, alors ([un ]) stationne en `, donc converge vers `.
 Si ∃n0 tq n ≥ n0 ⇒ un < `, alors ([un ]) stationne en ` − 1, donc converge vers ` − 1.
 Dans les autres cas, c’est-à-dire si pour tout N il existe n0 ≥ N et n1 ≥ N tels que un0 < `
et un1 ≥ ` alors la suite de terme ([un ]) n’est pas convergente (elle possède une suite
extraite convergeant vers ` et une autre convergeant vers ` − 1.)
(−1)n
Par exemple si un = , alors [u2n ] = 0 et [u2n+1 ] = −1.
n+1

Page 3 Jean-Michel Ferrard www.klubprepa.net EduKlub


c S.A.
Tous droits de l’auteur des œuvres réservés. Sauf autorisation, la reproduction ainsi que toute utilisation des œuvres autre que la consultation
individuelle et privée sont interdites.
Exercices de Mathématiques
Partie entière, Borne sup et inf
Corrigés

Corrigé de l’exercice 3 [ Retour à l’énoncé ]


Pour tout réel x, et tout entier k, on a kx ≤ [kx] < kx + 1.
x P n 1 Pn
On en déduit l’encadrement : 2 k ≤ un (x) < 2 (kx + 1).
n k=1 n k=1
n+1 n+1 1 x
Autrement dit : x ≤ un (x) < x + . On en déduit lim un (x) = .
2n 2n n n→+∞ 2

Corrigé de l’exercice 4 [ Retour à l’énoncé ]


 
x+k x+k
1. Soit q = . On a q ≤ < q + 1 c’est-à-dire qn − k ≤ x < (q + 1)n − k (1).
n n
Pour tous entiers m et p, on a : m ≤ x < p ⇐⇒ m ≤ [x] < p.
[x] + k
(1) équivaut donc à qn − k ≤ [x] < (q + 1)n − k, c’est-à-dire q ≤ <q+1 .
  n
[x] + k
On en déduit = q, ce qu’il fallait démontrer.
n
2. On commence par traiter le cas où x un entier relatif.
Soit x = qn + r la division euclidienne de x par n : le reste r vérifie 0 ≤ r ≤ n − 1.
     
x+k r+k r+k
Pour tout entier k de {0, . . . , n − 1}, on a : = q+ =q+ .
n n n
 
r+k
Remarquons que 0 ≤ r + k ≤ 2n − 2 < 2n : est donc égale à 0 ou 1.
n
 
r+k
– Si 0 ≤ k ≤ n − r − 1 (ce qui fait n − r possibilités) alors r + k ≤ n − 1 et = 0.
n
 
r+k
– Si n − r ≤ k ≤ n − 1 (ce qui fait r possibilités), alors r + k ≥ n et = 1.
n
On en déduit :
n−1   n−1    n−1  
X x+k X r+k X r+k
= q+ = nq +
k=0
n k=0
n k=0
n
n−r−1
X r + k  n−1  
X r+k
= nq + +
n
k=0 | {z }
n
k=n−r | {z }
=0 =1

= nq + (n − r) · 0 + r · 1 = nq + r = x
On suppose maintenant que x est un réel quelconque.
n−1   Xn−1  
X x+k [x] + k
D’après le (a), = .
k=0
n k=0
n
Mais [x] est un entier relatif, et cette dernière somme est égale à [x], comme on vient de
le voir. Le résultat est donc prouvé pour tout réel x.

Page 4 Jean-Michel Ferrard www.klubprepa.net EduKlub


c S.A.
Tous droits de l’auteur des œuvres réservés. Sauf autorisation, la reproduction ainsi que toute utilisation des œuvres autre que la consultation
individuelle et privée sont interdites.
Exercices de Mathématiques
Partie entière, Borne sup et inf
Corrigés

Corrigé de l’exercice 5 [ Retour à l’énoncé ]

– On note [x] la partie entière de tout réel x.


On commence par prouver l’unicité de la suite (an ), en supposant qu’elle existe.
n−1
P ak P ak
Pour simplifier, on note : sn = et rn = (cette dernière somme est finie).
k=2 k! k≥n+1 k!
an
Pour tout n ≥ 2, on a donc r = sn + + rn (avec la convention s2 = 0.)
n!
Si on note N un entier tel que n > N ⇒ an = 0 alors on a les inégalités :
N N
X ak X k−1
0 ≤ rn = ≤
k=n+1
k! k=n+1 k!
 
PN k−1 PN 1 1 1 1 1
Or = − = − < .
k=n+1 k! k=n+1 (k − 1)! k! n! N ! n!
On en déduit 0 ≤ n!rn < 1 puis an = n!(r − sn − rn ) ≤ n!(r − sn ) < an + 1.
Ainsi an est la partie entière de n!(r − sn ).
Plus précisément, an = [n!(r − sn )] = [n!r] − n!sn car n!sn est en entier.
On constate donc que la suite (an ), si elle existe, est définie de manière unique par :
 a2 = [2r] (ce qui implique 0 ≤ a2 ≤ 1 car 0 ≤ r < 1)
n−1
P ak
 Pour tout n ≥ 3, an = [n!(r − sn )] = [n!r] − n!sn = [n!r] − n! .
k=2 k!

– Il reste à prouver que la suite (an ) d’entiers définie par les conditions précédentes satisfait
aux conditions imposées par l’énoncé, c’est-à-dire :
 Pour tout n ≥ 2, on doit avoir 0 ≤ an ≤ n − 1.
 Il existe un entier N tel que n > N ⇒ an = 0.
P an
 La suite (an ) vérifie l’égalité r = .
n≥2 n!

On sait déjà que 0 ≤ a2 ≤ 1. On se donne donc un entier n ≥ 3.


Par construction, on a l’encadrement : an−1 ≤ (n − 1)!(r − sn−1 ) < an−1 + 1.
an−1 1 1
On en déduit : 0 ≤ r − sn−1 − < ou encore : 0 ≤ r − sn < .
(n − 1)! (n − 1)! (n − 1)!
Cette double inégalité s’écrit 0 ≤ n!(r − sn ) < n et elle implique 0 ≤ an ≤ n − 1.
Ainsi, pour tout entier n ≥ 2, on a : 0 ≤ an ≤ n − 1.
n−1
P ak
Il en découle en particulier que n!sn = n! est toujours un entier.
k=2 k!
Puisque r est rationnel, il existe un entier N tel que N !r soit entier.

Page 5 Jean-Michel Ferrard www.klubprepa.net EduKlub


c S.A.
Tous droits de l’auteur des œuvres réservés. Sauf autorisation, la reproduction ainsi que toute utilisation des œuvres autre que la consultation
individuelle et privée sont interdites.
Exercices de Mathématiques
Partie entière, Borne sup et inf
Corrigés

Pour tout n ≥ N , n!r et donc n!(r − sn ) sont des entiers.


On en déduit que pour tout n ≥ N , on a : an = n!(r − sn ).
Il en découle, pour tout n ≥ N :
h an i
an+1 = [(n + 1)!(r − sn+1 )] = (n + 1) · n!(r − sn − ) = 0
n!
Cela implique, toujours pour n ≥ N : 0 = an+1 = (n + 1)!(r − sn+1 ) = 0 donc sn+1 = r.
On a ainsi prouvé l’existence d’un entier N tel que : n > N ⇒ an = 0 et sn = r.
C’est ce qu’il fallait démontrer...

– Ce programme Maple calcule la liste des ak (jusqu’à N au sens précédent.)

> decomp :=proc(r : :rational)


local s,L,n,a ;
L :=[] ; s :=0 ;
for n from 2 do a :=floor(n !*(r-s)) ;
L :=[op(L),a] ;
if a=n !*(r-s) then break fi ;
s :=s+a/n ! ;
od ;
L;
end :
5
Voici la décomposition de r = .
7
> L :=decomp(5/7) ;
L := [1, 1, 1, 0, 4, 2]
5 1 1 1 4 2
On vérifie qu’on a bien = + + + + :
7 2! 3! 4! 6! 7!
> sum(L[k]/(k+1) !,k=1..nops(L)) ;
5
7

Page 6 Jean-Michel Ferrard www.klubprepa.net EduKlub


c S.A.
Tous droits de l’auteur des œuvres réservés. Sauf autorisation, la reproduction ainsi que toute utilisation des œuvres autre que la consultation
individuelle et privée sont interdites.
Exercices de Mathématiques
Partie entière, Borne sup et inf
Corrigés

Corrigé de l’exercice 6 [ Retour à l’énoncé ]

– Si t ≥ 0, l’application x 7→ x2 + tx est positive croissante sur [0, 1].


Dans ce cas sup |x2 + tx| est atteint pour x = 1 et vaut 1 + t ≥ 1.
x∈[0,1]

– Si t < 0, le tableau de variation de l’application x 7→ |x(x + t)| est le suivant :

On en déduit :
t
 Si 1 ≤ − , c’est-à-dire si t ≤ −2, alors : sup |x2 + tx| = |1 + t| ≥ 1.
2 x∈[0,1]

t t2
 Si − ≤ 1 ≤ −t, c’est-à-dire si −2 ≤ t ≤ −1, alors : sup |x2 + tx| = ≤ 1.
2 x∈[0,1] 4
1
(quantité décroissante par rapport à t, et de mimimum quand t = −1.)
4
t2
 Si −t ≤ 1, c’est-à-dire si −1 ≤ t ≤ 0, alors : sup |x2 + tx| = sup{ , |1 + t|} ≤ 1.
x∈[0,1] 4
t2 t2
Posons µ = inf ϕ(t), avec ϕ(t) = sup{ , |1 + t|} = sup{ , 1 + t}.
−1≤t≤0 4 4

t2

1 2 1 α = 2 − 2 2 ≈ −0, 83
Or − (1 + t) = (t − 4t − 4) = (t − α)(t − β), avec √ .
4 4 4 β =2+2 2>0
t2 t2
Ainsi on a ≥ 1 + t sur [−1, α] et ≤ 1 + t sur [α, 0].
4 4
On en déduit que :
t2
ϕ(t) = (fonction décroissante) sur [−1, α] et
4
ϕ(t) = 1 + t (fonction croissante) sur [α, 0].
√ 1
Finalement, µ = ϕ(α) = 1 + α = 3 − 2 2 < .
4
 Conclusion : La borne inférieure λ est obtenue en supposant −1 ≤ t ≤ 0.
√ √
Plus précisément, elle est obtenue quand t = 2 − 2 2 et elle vaut 3 − 2 2.

Page 7 Jean-Michel Ferrard www.klubprepa.net EduKlub


c S.A.
Tous droits de l’auteur des œuvres réservés. Sauf autorisation, la reproduction ainsi que toute utilisation des œuvres autre que la consultation
individuelle et privée sont interdites.

Vous aimerez peut-être aussi